PT38.S4.Q13: A recent study reveals that television advertising does not

redshiftredshift Alum Member
edited November 2018 in Logical Reasoning 261 karma

I understand why (A) is correct, as well as why the wrong answers are incorrect. That being said, I'm having trouble identifying the type of flaw/assumption that this argument is making.

Is this a study flaw? Is the author assuming that the data from this study is solid enough to prove that there's no causal link between tv advertising and cereal preferences?

Usually, when I attempt weaken questions, I try and identify the type of flaw/assumption that the argument is making. Here, I couldn't do so, and just had to get to the right answer through process of elimination. When I inserted answer choice (A) into the premises, I saw how it weakened the conclusion, but I still don't know which type of flaw this question falls into.

Comments

  • redshiftredshift Alum Member
    edited November 2018 261 karma

    Actually, I figured it out. This is an Experiment to reach a General Conclusion flaw. Essentially, the author is concluding something definitive (that tv has no impact on cereal preferences) from the results of an experiment.

    Of course, in making this argument, the author takes a lot of things for granted. Namely, that the experiment is solid enough to grant the conclusion. Of course, we know that this may not be true. The experiment may not be representative of an entire population (perhaps both groups were taken from areas in the country that love sugary cereals), the two groups in the experiment could've been flawed (perhaps the group that watched no television actually watched a lot of television prior to the experiment), etc. In essence, the author is taking for granted that the experiment isn't flawed in some way. Note that in the stimulus, we're given no information that this is a carefully conducted experiment, that all variables have been accounted for, etc. This leaves open the possibility for an experimental flaw.

    To weaken an LR question, we want to attack the assumption. Here, we can do this by showing that the experiment is flawed in some way.

    (A) - Bingo. The experiment is flawed because, if kids who don't watch tv are influenced by kids who do, then the two groups mentioned in the study aren't as distinct as such an experiment like this one requires. It's actually the case that we're not studying two groups, one of which wasn't impacted by tv and another that was, but rather two groups where one was impacted by tv advertising directly and another tangentially.

  • M.Yanka106M.Yanka106 Member
    113 karma

    Hi,

    I categorized this question as cause-effect (with a weak conclusion "does not significantly affect"). In this case, no-cause-effect. The argument claims that advertising does not significantly affect children's preference. You can weaken this argument by showing a way in which TV can influence. Ans A does that.

Sign In or Register to comment.